PT44.S2.Q22 - All the evidence so far gathered fits both ...

Really confused by the extent to which we rely on diagram in this question. For PF questions, when is the case that we do not strictly follow the diagram in the stimulus? https://7sage.com/lsat_explanations/lsat-44-section-2-question-22/

STIMULUS:

H–>/G
——-
H /G (confirm one theory at the expense of the other)

However, AC E:

D–>/J
——–
/D–>J (either or)

If the above is true, why is AC E is still the correct AC? Or in this case, are we choosing the best AC?

Sign In or Register to comment.